ACCP PHARMACOTHERAPY EXAM NEWEST 2025
ACTUAL EXAM TEST BANK| COMPLETE 350 REAL
EXAM QUESTIONS AND CORRECT DETAILED
ANSWERS (VERIFIED ANSWERS WITH WELL
ELABORATED RATIONALES) GRADED A+|| NEW!!
(TO STUDY FOR THE BCPS EXAM)
A 76-year-old woman presents to the clinic with mild to moderate right
knee pain related to osteoarthritis (OA). Her medical history is also
significant for HTN, coronary artery disease, and hyperlipidemia. Her
home drugs include carvedilol 25 mg twice daily, atorvastatin 20 mg
daily, aspirin 81 mg daily. nitroglycerin 0.4 mg sublingually as needed
for chest pain, and isosorbide dinitrate extended release 20 mg twice
daily. Ranolazine 500 mg twice daily was just added to therapy regimen
because of continued angina symptoms, patient's vital signs today
include blood pressure 122/72 mm Hg and heart rate 66 beats/minute.
She is allergic to penicillin and sulfonamides.
Which of the following is the best recommendation for her OA-related
pain?
A. Acetaminophen 500 mg every 4 hours,
B. Ibuprofen 800 mg every 4 hours.
C. Celecoxib 200 mg twice daily.
D. Tramadol 100 mg every 6 hours. - Correct Answer - A:
Acetaminophen 500 mg every 4 hours.
(According to the American College of Rheumatology osteoarthritis
guidelines, acetaminophen should considered as first-line therapy for the
management of mild to moderate knee or hip OA (Answer A is correct).
pg. 1
,2|Page
Although nonselective NSAIDs are also effective for OA, the dose of
ibuprofen exceeds the maximal daily dose of 3200 mg and should be
avoided because of the risk of cardiovascular events in a patient with
controlled disease (Answer B is incorrect). Celecoxib can be used in
dosages of up to 400 mg/day; however, it should be avoided in patients
with poorly controlled coronary artery disease and in those allergic to
sulfonamides (Answer C is incorrect).
Tramadol is indicated for moderate to moderately severe pain and should
used at doses Of no more than 300 mg/day in patients older than 75
years (Answer D is incorrect).)
A 76-year-old woman has bilateral knee OA pain that has not been
sufficiently controlled with physical therapy, simple analgesics, systemic
NSAIDs, or a short trial of opioid combination analgesics. Today, she
presents to the clinic for follow-up. She is unable to perform many
activities of daily living because she requires a walker, which
considerably impairs her mobility.
Which one of the following is the best long-term therapy option to
address this patient's chronic pain?
A. Glucosamine 1500 mg and chondroitin 1200 mg/day.
B. Ketorolac 10 mg every 6 hours.
C. Morphine sulfate extended release 15 mg twice daily.
D. Intra-articular corticosteroid injection up to every 3 months as
needed. - Correct Answer - D: Intra-articular corticosteroid injection up
to every 3 months as needed.
(Intra-articular corticosteroids are the best choice to help alleviate this
patient's chronic pain secondary to her OA of the knee. Although the
pg. 2
,3|Page
medication may take several days to reach maximal efficacy, it will offer
her the most long-term pain relief with fewer systemic adverse reactions
(Answer D is correct).
According to the American College of Rheumatology guidelines for the
treatment of OA, glucosamine and chondroitin are not recommended for
patients with OA of the knee (Answer A is incorrect).
Ketorolac will provide the most significant pain relief in the least
amount of time, but use beyond 5 days increases the risk of
gastrointestinal bleeding: thus, ketorolac is not a long-term option for
this patient (Answer B is incorrect).
Finally, the use of opioids for OA pain (and in the elderly) should
discouraged of the increased of somnolence and unsteady gait (Answer
C is incorrect).)
A 35-year-old woman has had rheumatoid arthritis (RA) for 10 years;
her disease has been in remission for 6 months. She currently takes
methotrexate 7.5 mg weekly and etanercept 50 mg weekly. The patient
now states that she wants to become pregnant and asks about continuing
her RA treatment during pregnancy.
Which one of the following is the best recommendation for this patient?
A. Discontinue methotrexate and continue etanercept.
B. Discontinue methotrexate and etanercept and begin leflunomide.
C. Discontinue etanercept and continue low-dose methotrexate.
D. Discontinue all drugs. - Correct Answer - Answer A: Discontinue
methotrexate and continue etanercept.
pg. 3
, 4|Page
(Patients with long-standing RA are likely to need maintenance therapy
with disease-modifying antirheumatic drugs or biologics, even after
achieving a short-term remission. Thus, discontinuing methotrexate, a
known teratogen, to allow a possible pregnancy is reasonable while
continuing etanercept, which largely appears safe in pregnancy (Answer
A is correct; Answer C is incorrect).
It would inappropriate to discontinue all drug treatment at once because
the patient would likely suffer a flare of her RA symptoms (Answer D is
incorrect).
Substituting leflunomide would be inappropriate because it is also
teratogenic (Answer B is incorrect).)
A 63-year-old man with HTN is referred to you for cardiovascular risk
reduction, He takes lisinopril 20 mg daily and furosemide 20 mg daily.
His BP is 135/78 mm Hg. Pertinent laboratory values include K 4.3
mEq/L and SCr 1.0 mg/dL. Fasting lipid profile is TC mg/dL, LDL-C
126 mg/dL, HDL-C 49 mg/dL, and TG 145 mg/dL. Using the Cohort
Equation, his 10-year risk of atherosclerotic cardiovascular disease
(ASCVD) is 14.3%.
Which one of the following is to recommend for this patient?
A. Ezetimibe 10 mg daily.
B. Simvastatin 10 mg daily.
C. Pravastatin 20 mg daily.
D. Rosuvastatin 10 mg daily - Correct Answer - D: Rosuvastatin 10 mg
daily.
(According to the recent guideline recommendations, this meets the
criteria for a statin-benefitting group (i.e., 40-75 years of age with LDL-
pg. 4